Two markers A and B on the same side of a canyon rim are 56 feet apart. A third marker C, located across the rim. is positioned so that BAC = 69º and ABC = 51° Complete parts (a) and (b) below (a) Find the distance between C and A.

Answers

Answer 1

To answer this question, it will be helpful to have a drawing of the situation to find the asked distance:

With this information, it will be easier to have all the information to solve for the distance CA.

Therefore, to find the distance CA, we can apply the Law of Sines, in which we have to find the angle C. We know that the sum of the interior angles of a triangle is equal to 180. Then, we have:

[tex]mNow, we can apply the Law of Sines to find the distance CA:[tex]\frac{AC}{\sin(51)}=\frac{56}{\sin(60)}\Rightarrow AC=\frac{56\cdot\sin (51)}{\sin (60)}[/tex]

Then, we have:

[tex]AC=50.2527681652ft[/tex]

Then, to round to one decimal place, we have that AC is approximately 50.3 ft.

To find the distance between the two rims, we have:

Now, we can also apply the Law of Sines to find the distance CD (the distance between the two rims):

[tex]\frac{CD}{\sin(69)}=\frac{CA}{\sin(90)}\Rightarrow CD=CA\cdot\sin (69),\sin (90)=1[/tex]

Then, we have:

[tex]CD=50.2527681652\cdot\sin (69)\Rightarrow CD=46.9150007363ft[/tex]

Therefore, the distance between the two canyon rims (round to one decimal place) is 46.9 ft.

If we take 50.3 ft (for CA), instead, we have 47 ft.

Two Markers A And B On The Same Side Of A Canyon Rim Are 56 Feet Apart. A Third Marker C, Located Across
Two Markers A And B On The Same Side Of A Canyon Rim Are 56 Feet Apart. A Third Marker C, Located Across
Two Markers A And B On The Same Side Of A Canyon Rim Are 56 Feet Apart. A Third Marker C, Located Across
Two Markers A And B On The Same Side Of A Canyon Rim Are 56 Feet Apart. A Third Marker C, Located Across

Related Questions

If a1 = 8 and an = 3an-1 then find the value of a4.

Answers

a_4= 216

1) Given that we have the first term and the Recursive Formula, let's find the fourth term of that Sequence

2) Let's find the second, the third to find the fourth since a Recursive formula depends on the prior term.

[tex]\begin{gathered} a_1=8 \\ a_n=3a_{n-1} \\ a_2=3(8)\text{ =24} \\ a_3=3(24)=72 \\ a_4=3(72)\text{ =216} \end{gathered}[/tex]

3) Hence, the sequence is 8, 24, 72, 216 and the fourth term is 216

A product initially with a value of $21,800 has been depreciating at 8.1% p.a over 8 years. What is it's current value?

Answers

we get that:

[tex]v=21800\cdot(0.919)^8=11091.25[/tex]

its current value is $11091.25

1.At 8:30am, Student Life had served 37 meals at their pancake breakfast. By 10:15am, thetotal served had reached 77. Find the serving rate, in number of meals per minute. Keep youranswer as a reduced fraction.

Answers

We have to estimate a serving rate, with units of "number of meals per minute".

To solve this we have to calculate how many meals have been served in a certain time frame, and estimate a mean service rate.

The time interval we are taking is from 8:30 am to 10:15 am. That is 30 minutes to 9 am, plus 60 minutes to 10 am, plus 15 minutes to 10:15 am.

This is a total of 30+60+15=105 minutes.

The total served at 10:15 am is 77 meals. If we substract the meals that have been already served by 8:30 am, we get that in our time interval 77-37=40 meals have been served.

So we can calculate the serving rate as:

[tex]s=\frac{\text{ \#meals}}{\text{time}}=\frac{40\text{ meals}}{105\text{ min}}=\frac{8}{21}\text{ meals/min}[/tex]

The serving rate, expressed as a reduced fraction (we divide both numerator and denominator by 5), is 8/21 meals per minute.

Members of the football team hold a fundraising dinner to raise money for their annual trip. They must sell tickets to the event at a price that will earn them more money than the cost of food.Here's a formula for this scenario:t = n (p - c)wheret = total profit made from the eventn = number of tickets soldp = price charged for each dinnerC = cost for food per plate The team hopes to sell 100 tickets. The cost for food per plate is $1.75 and they hope to charge $11.75 for each dinner. How much profit should they receive from the event?Enter the correct answer.

Answers

t = n(p-c)

t=100(11.75 - 1.75)

t = 100(10)

t=$1000

total profit received = $1000

The weekly revenue for a product is given by R(x)=307.8x−0.045x2, and the weekly cost is C(x)=10,000+153.9x−0.09x2+0.00003x3, where x is the number of units produced and sold.(a) How many units will give the maximum profit?(b) What is the maximum possible profit?

Answers

Answer:

The number of units that will give the maximum profit is;

[tex]1900\text{ units}[/tex]

The maximum possible profit is;

[tex]\text{ \$}239,090[/tex]

Explanation:

Given that the weekly revenue for a product is given by ;

[tex]R(x)=307.8x-0.045x^2[/tex]

and the weekly cost is ;

[tex]C(x)=10,000+153.9x-0.09x^2+0.00003x^3[/tex]

Recall that

Profit = Revenue - Cost

[tex]P(x)=R(x)-C(x)[/tex][tex]\begin{gathered} P(x)=307.8x-0.045x^2-(10,000+153.9x-0.09x^2+0.00003x^3) \\ P(x)=307.8x-0.045x^2-10,000-153.9x+0.09x^2-0.00003x^3 \\ P(x)=153.9x+0.045x^2-0.00003x^3-10,000 \end{gathered}[/tex]

Using graph to derive the maximum point on the function;

Therefore, the maximum point is at the point;

[tex](1900,239090)[/tex]

So;

The number of units that will give the maximum profit is;

[tex]1900\text{ units}[/tex]

The maximum possible profit is;

[tex]\text{ \$}239,090[/tex]

Examine the following graph of the system of inequalities y≤x2−4x−3 and y<−2x+4. A is the area below the line and the parabola. B is the area below the line but above the parabola. C is the area above the line and the parabola. D is the area below the parabola but above the line.© 2018 StrongMind. Created using GeoGebra. Which section of the graph represents the solution set to the system of inequalities?

Answers

The solution set to a system of inequalities represents the area that contains points that satisfy both inequalities.

The best way to answer the question is to choose one point from each area and check if they satisfy both.

Let's start by selecting a point in area A. Let's use (-6, 0).

[tex]\begin{gathered} 0\leq(-6)^2-4(-6)-3 \\ 0\leq36+24-3 \\ 0\leq57\text{ TRUE} \\ \\ 0<-2(-6)+4 \\ 0<12+4 \\ 0<16\text{ TRUE} \end{gathered}[/tex]

Because out test point (-6, 0) satisfies both inequalities, then the entire area that contains it is the solution. We no longer have to test other points.

The answer is A.

The solution set is also the intersection of the graphs of the two inequalities. So you may refer to the shaded regions and you'll see that area A is shaded red and blue at the same time.

the perimeter of a rectangle room is 60 feet. let x be the width of the room (in feet) and let y be the length of the room (in feet). select all of the questions below that could modle this situation

Answers

Given that,

The perimeter of a rectangle is 60.

The perimeter is generally defined as the length of the outline of the shape.

So, in rectangle having four sides, the perimeter would be sum of all the sides.

Length1 + length2 + length3 + length4 = perimeter

Here, length1 and length3 are equal, that are the lengths (y),

Similarly,

Length2 and length4 are equal, that is width (x).

Hence, the equation becomes,

x + y + x + y = perimeter

or

2x + 2y = 60

or

2 (x + y) = 60

Hence, the first two options are correct.

Zappo’s has marked down rain boots 25% during its spring sale. What is the sale price of a pair of boots with a regular price of $149.99?

Answers

Substract the 25% of the price to the regular price:

1. Find the 25% of the price: Multiply by 0.25 the price:

[tex]149,99\cdot0.25\approx37.50[/tex]

2. Substract the result in step 1 (amount marked down) from the regular price:

[tex]149,99-37,50=112.49[/tex]Then, the sale price of the pair of boots is $112.49

For what value of k are the graphs of 8y = 12x + 6 and 4y = k(3x + 10)
parallel? perpendicular?

Answers

The value of k when the graphs are parallel = 2 and when the graphs are  perpendicular = 36/32 or 1.125

What is the slope-intercept form?

the slope-intercept for of a line is, y=mx+c, where m is the slope.

we are given the two equations  8y = 12x + 6 and 4y = k(3x + 10)

PARALLEL CONDITION

if they are parallel, their slopes will be equal,

hence,

y=mx+c

where m is the slope

converting both the equations in the slope-intercept form

8y = 12x + 6

= y= 12x/8 + 3/4

and for equation

4y = k(3x + 10)

4y = k3x+ 10k

y = k 3x/4 + 10k/4

comparing the slopes

12/8 = k3/4

12 * 4 = 3k * 8

48 = 24k

k= 48/24

k = 2

therefore when both the lines are parallel, the value of k is 2.

PERPENDICULAR CONDITION

if the two lines are perpendicular the product of their slope will be 1

so,

12/8 * 3k/4 = 1

36k/32 =1

36k = 32

k = 32/36

or

k = 1.125

therefore when they both are perpendicular, the value of k is 1.125 or 36/32.

to learn more about slope-intercept for visit:

https://brainly.com/question/28790743

#SPJ13

Question 13Triesremaining: 3Check for UnderstandingPoints out of2.00Write the equation of the line with a slope of 10 that goes through the point(8,-2) in Slope-Intercept and Point-Slope Form.P FlagquestionSlope-Intercept Form:Point-Slope Form:CheckPrevious page

Answers

[tex]y=10x-82\rightarrow slope\text{ intercep form}[/tex][tex]y+2=10x-8\rightarrow Point-SlopeForm[/tex]

Explanation

Step 1

Let

passes throught (8,-2)

slope=20

then

a)slope -intercept form

[tex]y-y_1=m(x-x_1)[/tex]

where

[tex]\begin{gathered} (x_1,y_1)\text{ is a point of the line} \\ m\text{ is the slope of the line} \end{gathered}[/tex]

replace

[tex]\begin{gathered} y-y_1=m(x-x_1) \\ y-(-2)=10(x-8) \\ y+2=10x-80 \\ \text{subtract 2 in both sides} \\ y+2-2=10x-80-2 \\ y=10x-82\rightarrow slope\text{ intercep form} \end{gathered}[/tex]

Step 2

Now, Point-Slope Form, use the same equation, but don't isolate y

[tex]\begin{gathered} y-y_1=m(x-x_1) \\ y-(-2)=10(x-8) \\ y+2=10x-8\rightarrow Point-SlopeForm \end{gathered}[/tex]

To which subsets of numbers does 1/3 belong?

Answers

1/3 is a rational number, which written as a decimal is an infinite period decimal.

Using the translation that maps (3,-4) to its image (1,0), what is the image of any point (x,y)?A. (x+2,y+4)B. (x−2,y−4)C. (x+2,y−4)D. (x−2,y+4)

Answers

Explanation

Given (3,-4), its image (1,0) can be produced below;

[tex]\left(3,-4\right)\Rightarrow\left(x−2,y+4\right)\Rightarrow\left(1,0\right)[/tex]

Answer: Option D

miguel saves the same amount of money into a bank account each week. the bank account started with some money in it. after 3 weeks, the bank account contained $250. after 10 weeks the bank account contained $600.write an equation that cqn be used to model tbe number of dollars, y,iguel saves in x weeks.exain what slope and y intercept of youre equation mean in the context of the aituation.enter your equation and your explanations in tbe space provided.

Answers

Answer:

An equation that can be used to model the number of dollars, y, Miguel saves in x weeks is;

[tex]y=50x+100[/tex]

The slope of the equation in the context is the amount of money Miguel saves in the bank account each week. So, Miguel saves $50 each week.

The y-intercept of the equation in the context is the amount of money Miguel initially have in the bank account. So, the initial amount of money in the bank account is $100.

Explanation:

Given that Miguel saves the same amount of money into a bank account each week.

Let y represent the amount of money in the account after x weeks;

[tex]y=mx+b[/tex]

After 3 weeks, the bank account contained $250;

[tex]\begin{gathered} 250=m(3)+b \\ 3m+b=250 \end{gathered}[/tex]

After 10 weeks the bank account contained $600;

[tex]\begin{gathered} 600=m(10)+b \\ 10m+b=600 \end{gathered}[/tex]

Solving for m and b;

subtract the first equation from the second.

[tex]\begin{gathered} 10m-3m+b-b=600-250 \\ 7m=350 \\ m=\frac{350}{7} \\ m=50 \end{gathered}[/tex]

substituting the value of m into the first equation;

[tex]\begin{gathered} 3m+b=250 \\ 3(50)+b=250 \\ 150+b=250 \\ b=250-150 \\ b=100 \end{gathered}[/tex]

Therefore, an equation that can be used to model the number of dollars, y, Miguel saves in x weeks is;

[tex]y=50x+100[/tex]

From the equation above, the slope m of the equation is;

[tex]m=50[/tex]

and the y-intercept b of the equation is;

[tex]b=100[/tex]

The slope of the equation in the context is the amount of money Miguel saves in the bank account each week. So, Miguel saves $50 each week.

The y-intercept of the equation in the context is the amount of money Miguel initially have in the bank account. So, the initial amount of money in the bank account is $100.

Select the correct answer.6cis5pi/6Convert57to rectangular form.OA. 3V3 + 31O B. –313 + 3iO C. 373 – 3iOD. -3V3 – 31O E. 3 – 3731

Answers

Answer:

Choice B.

Explanation:

The equation can be rewritten as

[tex]6\text{cis}\frac{5\pi}{6}=6\cos \frac{5\pi}{6}+i\sin \frac{5\pi}{6}[/tex]

Now since

[tex]6\cos \frac{5\pi}{6}=-3\sqrt[]{3}[/tex]

and

[tex]6\sin \frac{5\pi}{6}=3[/tex]

the expression becomes

[tex]-3\sqrt[]{3}+3i[/tex]

Hence, choice B is the correct answer since it matches the answer we got above.

Use the system of equations below to solve for z.7x+3y+2z-4w=184w+5x-3y-2z=6-2w-3x+y+z=-52z+3w+4y-8x=11253

Answers

Equations:

[tex]\begin{gathered} 7x+3y+2z-4w=18\text{ \lparen1\rparen} \\ 5x-3y-2z+4w=6\text{ \lparen2\rparen} \\ -3x+y+z-2w=-5\text{ \lparen3\rparen} \\ -8x+4y+2z+3w=11\text{ \lparen4\rparen} \end{gathered}[/tex]

Sum (1)+ (2):

[tex]\begin{gathered} 7x+3y+2z-4w=18\text{ }\operatorname{\lparen}\text{1}\operatorname{\rparen} \\ + \\ 5x-3y-2z+4w=6\text{ }\operatorname{\lparen}\text{2}\operatorname{\rparen} \\ 5x+7x+3y-3y+2z-2z-4w+4w=18+6 \\ 12x=24 \\ x=\frac{24}{12}=2 \end{gathered}[/tex]

x=2

Now, we are going to sum (3)*2+(2).

[tex]\begin{gathered} 5x-3y-2z+4w=6\text{ }\operatorname{\lparen}\text{2}\operatorname{\rparen} \\ + \\ 2*(-3x+y+z-2w)=-5*2\text{ }\operatorname{\lparen}\text{3}\operatorname{\rparen} \\ 5x-6x-3y+2y-2z+2z+4w-4w=6-10 \\ -x-y=-4 \\ -2-y=-4 \\ y=-2+4=2 \end{gathered}[/tex]

y=2.

Replacing y and x in (4) and (3):

[tex]\begin{gathered} -3(2)+2+z-2w=-5\text{ }\operatorname{\lparen}\text{3}\operatorname{\rparen} \\ -8(2)+4(2)+2z+3w=11\text{ }\operatorname{\lparen}\text{4}\operatorname{\rparen} \end{gathered}[/tex][tex]\begin{gathered} -6+2+z-2w=-5 \\ z-2w=-5+6-2 \\ z-2w=-1\text{ \lparen5\rparen} \end{gathered}[/tex][tex]\begin{gathered} -16+8+2z+3w=11 \\ 2z+3w=11+16-8 \\ 2z+3w=19\text{ \lparen6\rparen} \end{gathered}[/tex]

Isolating w in (5) ans replacing in (6):

[tex]\begin{gathered} 2w=-1-z \\ w=\frac{-1-z}{2} \end{gathered}[/tex][tex]\begin{gathered} 2z+3(\frac{-1-z}{2})=19 \\ \frac{4z-3-3z}{2}=19 \\ z-3=19*2 \\ z=38-3=35 \end{gathered}[/tex]

Answer: z=35.

Please help me name this figure, find the lateral surface area, and the total surface area. You can ignore the work I've done as it is incorrect.

Answers

Answer:

• (,a)Triangular Prism

,

• (b)Lateral Surface Area= 36 cm²

,

• (c)Total Surface Area= 48 cm²

Explanation:

(a)The figure has a triangle as its uniform cross-section. Thus, it is a triangular prism.

(b)Lateral Surface Area

The lateral surface area is the area of the sides of the prism, i.e. excluding the uniform top and base.

The sides of the triangular prism consist of the three rectangles.

[tex]\begin{gathered} \text{Lateral Surface Area}=\text{Area of Rect. 1+Area of Rect. 2+Area of Rect. 3} \\ =(3\times4)+(3\times3)+(3\times5) \\ =12+9+15 \\ =36\;cm^2 \end{gathered}[/tex]

The lateral surface area is 36 cm squared.

(c)Total Surface Area

To find the total surface area, add the area of the top and base to the lateral surface area.

The top and base are the two right-triangles with a base of 3 cm and a height of 4cm.

[tex]\begin{gathered} \text{ Total Surface Area=Lateral Surface Area+2\lparen Area of Triangles\rparen} \\ =36+2(\frac{1}{2}\times3\times4) \\ =36+12 \\ =48\;cm^2 \end{gathered}[/tex]

The total surface area is 48 cm squared.

The blank of a line is the x-coordinate of the point where the line crosses the x-axis. It occurs when y = 0.

Answers

Answer

The x-intercept of a line is the x-coordinate of the point where the line crosses the x-axis. It occurs when y = 0.​

Hope this Helps!!!

What is the radius of a circle whose circumference is 36pi?

Answers

The circumference of a circle of radius r is given by:

[tex]C=2\pi r[/tex]

For this question we simply need to take C=36π and solve for r:

[tex]36\pi=2\pi r[/tex]

If we divide both sides by 2π we get:

[tex]\begin{gathered} 36\pi=2\pi r \\ \frac{36\pi}{2\pi}=\frac{2\pi r}{2\pi}=\frac{2\pi}{2\pi}\cdot r \\ 18=r \end{gathered}[/tex]

Then the answer is option A, 18.

Solve x² + 6x + 7 = 0.x = -1 and x = -5 3+ √2-3+√2-3 ± √22

Answers

ANSWER

[tex]x=-3\pm\sqrt{2}[/tex]

EXPLANATION

We want to solve for x in the equation:

[tex]x^2+6x+7=0[/tex]

To do this, apply the quadratic formula:

[tex]x=\frac{-b\pm\sqrt{b^2-4ac}}{2a}[/tex]

where a = 1, b = 6, c = 7

Therefore, substituting the values of a, b, and c into the formula and solve:

[tex]\begin{gathered} x=\frac{-6\pm\sqrt{6^2-4(1)(7)}}{2(1)}=\frac{-6\pm\sqrt{36-28}}{2} \\ \\ x=\frac{-6\pm\sqrt{8}}{2}=\frac{-6\pm2\sqrt{2}}{2} \\ \\ x=-3\pm\sqrt{2} \end{gathered}[/tex]

That is the solution for x.

scientific notation5.1x10⁶ x 2.3x10⁶

Answers

The given expression is

5.1 x 10^6 x 2.3 x 10^6

We would apply the law of exponents which is expressed as

a^b x a^c = a^(b + c)

By applying this, we have

5.1 x 2.3 x 10^6 x 10^6

= 11.73 x 10^(6 + 6)

= 11.73 x 10^12

A girl has scored 72, 76, 74, and 75 on her algebra tests.a. Use an inequality to find the score she must make on the final test to pass the course with an average of 78 or higher, given that the final exam counts three testsb. Explain the meaning of the answer to part (a)

Answers

Part A.

We know that the final exam counts 3 test. Let x be the score of this final exam, then we can write,

[tex]\frac{72+76+74+75+3\times x}{7}\ge78[/tex]

the denominator is 7 because there are 7 scores: 72,76,74,75 and 3 times x (which is the final test). This average must be greater or equal to 78 in order to pass the course.

Then, by adding the numerator terms we get,

[tex]\frac{297+3\times x}{7}\ge78[/tex]

and by moving 7 to the right hand side, we have

[tex]\begin{gathered} 297+3x\ge78\times7 \\ or\text{ equivalently} \\ 297+3x\ge546 \end{gathered}[/tex]

by moving 297 to the right hand side, we obtain

[tex]\begin{gathered} 3x\ge546-297 \\ 3x\ge249 \end{gathered}[/tex]

then, the score of the final exam must be

[tex]\begin{gathered} x\ge\frac{249}{3} \\ x\ge83 \end{gathered}[/tex]

that is, at least greater or equal to 83.

Part B.

The last result means that our girls must obtain at least 83 points in the final test in order to pass Algebra.

Part #1: Find the solution of the inequality.[tex]n - 6 \ \textgreater \ 10[/tex]Part #2: describe the solution

Answers

[tex]n-6>10\rightarrow n>10+6\rightarrow n>16[/tex]

so the solution is all the numbers that are greater than 16

There are 396 students who are enrolled in an introductory engineering course. If there are four boys to every seven girls, how many boys are in the course?

Answers

Solution

For this case we know that the total of students are 396 so we can do this:

x + y = 396

Where:

x= number of girls

y = number of boys

Then we have the following condition:

4x = 7y

Then solving for x we got:

x = 7/4 y

Replacing in the first equation we got:

7/4 y + y = 396

11/4 y= 396

y= 396*4/11 = 144

And x= 7/4 * 144 = 252

Then the answer would be:

252 girls and 144 boys

What is the measure of the "Central Angle" for the 20% section?

Answers

The sum of all central angle is 360.

Determine 20% of 360 to obtain central angle for 20% section.

[tex]\begin{gathered} \frac{20}{100}\times360^{\circ}=36^{\circ}\cdot2 \\ =72^{\circ} \end{gathered}[/tex]

So answer is 72 degrees.

This is a maze where you find the answer starting from where it says start, and as you find the answer you highlight it along the way! Pls help I’m really bad at this

Answers

The start figure has two chords in the circle.

By theorem of internal division of chords it follows:

[tex]\begin{gathered} 21x=18\times14 \\ x=\frac{18\times14}{21} \\ x=12 \end{gathered}[/tex]

Hence the value of x is 12.

Every surface of the block shown will be painted, except for one of the bases. How many square units will be painted? 3.2 cm 3.2 cm 3.2 cm

Answers

As given that the every surface of the block shown will be painted, except for one of the bases.

And a block have 6 surface but one is not painted so there are 5 surface that are painted so:

The area of the painted surface is:

[tex]A=5a^2[/tex]

Where a is 3.2 cm

[tex]\begin{gathered} A=5(3.2)^2 \\ A=5\times10.24 \\ A=51.2cm^2 \end{gathered}[/tex]

The area of painted surface is 51.2 square cm

1. if m∠6 =50° , then find m∠112. m∠2= 70°, then find m∠63. if m∠ 1=130°, then find m∠5

Answers

Answer:

1. m∠11=130°

2. m∠6= 70°

3. m∠5=130°

Explanation:

Part 1

Angles 6 and 11 are the same-side interior angles. We know that same side interior angles add up to 180 degrees, therefore:

m∠6+m∠11=180°

50°+m∠11=180°

m∠11=180°-50°

m∠11=130°

Part 2

Lines a and b are parallel lines. Therefore, angles 2 and 6 form a Z-Shape.

They are Alternate angles.

m∠2 = m∠6

Since m∠2= 70°

m∠6= 70°

Part 3

Angles 1 and 3 are Corresponding angles, this means that they are equal.

• m∠1=m∠3

Likewise, angles 3 and 5 form an X-shape, they are vertical angles and also equal.

• m∠5=m∠3

Combining the two, we have that:

m∠1=m∠3=m∠5

If m∠1=130°, then:

m∠5=130°

Solve: 9/14 + 2/6 = ?

Answers

We have to solve the expression:

[tex]\frac{9}{14}+\frac{2}{6}[/tex]

We have to find a common denominator for the fractions and then solve it.

We can start by simplifying the fractions that can be simplified, like 2/6.

[tex]\frac{9}{14}+\frac{2}{6}=\frac{9}{14}+\frac{1}{3}[/tex]

Then, the common denominator between 14 and 3 is 14*3=42, so we end with:

[tex]\frac{9\cdot3}{14\cdot3}+\frac{1\cdot14}{3\cdot14}=\frac{27}{42}+\frac{14}{42}=\frac{27+14}{42}=\frac{41}{42}[/tex]

Answer: 41/42

This expression 12(1.0515)t models the population of elephants in a wildlife refuge after years since 1975 is the population of elephants increasing or decreasing?

Answers

The function for an exponential growth/decay is given as follows;

[tex]f(x)=a(1+r)^x[/tex]

Where,

[tex]\begin{gathered} x=\text{Number of years} \\ a=\text{initial value} \\ r=\text{rate of growth} \end{gathered}[/tex]

Observe that from the equation provided, the rate is 1.015. This means there is a growth. If there was a decay(decrease), the rate would be less than 1 because, the formula then would be;

[tex]f(x)=a(1-r)^x[/tex]

ANSWER:

Therefore, the population of elephants is INCREASING.

Rosa sells cosmetics. She is paid a commission of 3.16% of her first 1500 in sales during the week and 11% on all sales over 1500. What is her commission in a week during which she sells 2137.38 worth of cosmetics? Express your answer as a dollar amount and round to the nearest cent

Answers

ANSWER:

$ 117.51

STEP-BY-STEP EXPLANATION:

The commissions are divided into two payments, the first payment of the first $ 1500 with a commission of 3.16% and a second payment with a commission of 11% of all the remaining money of the first $ 1500.

Therefore, we calculate it as follows:

[tex]\begin{gathered} p_T=p_1+p_2 \\ p_1=1500\cdot\frac{3.16}{100}=47.4 \\ p_2=(2137.38-1500)\cdot\frac{11}{100}=637.38\cdot0.11=70.11 \\ p_T=47.4+70.11 \\ p_T=117.51 \end{gathered}[/tex]

The total commission is $ 117.51

Other Questions
Need help with this thanks! The first equation is 4x-3 Find the slope between the given points and write an equation in slope-intercept form. (2, -9) and (8, -6) The safe load, L, of a wooden beam of width w, height h and length l, supported at both ends, varies directly as the product of the width and the square of the height and inversely as the length. A wooden beam 5 inches wide, 7 inches high and 144 inches long can hold a load of 8740 pounds. What load would a beam 6 inches wide, 9 inches high, and 216 inches long of the same material, support? Round your answer to the nearest integer if necessary. During which era did Pangaea break up?A. CenozoicB. MesozoicC. PaleozoicD. Precambrian Which of the following does a polygraph measure?AnxietySympathetic arousalFearDeceit Having a hard time explaining to my daughter how to explain her estimate of this problem. Which is the smallest ratio? 2 3 to 4, 3, 10:12, 2 to 1 03 to 4 em O 10:12 2 to 1 the next dividend payment by hoffman, inc., will be $2.85 per share. the dividends are anticipated to maintain a growth rate of 5.00 percent forever. if the stock currently sells for $49.30 per share, what is the required return? (do not round intermediate calculations and round your answer to 2 decimal places, e.g., 32.16.) 15/9 equals 40 over n what helps create metamorphic and sedimentary rock I mostly need to know if this are correct and if the answers would gave been affected. are these equivalent 12:8 and 18:12 a bank loaned out 2000, part of it at the rate of 8% per year and the rest at 16% per year. If the interest received in one year totaled $2000, how much was loaned at 8%? Simplify the equation 3( m- 5 ) + m textFor this fraction 12/13 the numerator is The population of a certain species of owl at a wildlife preserve can beapproximated by the functionN(t) =20401+39e-0.51where N(t) represents the number of owls and t is the time (in years).a.) What was the initial population of the owls?b.) How many owls will there be in the wildlife preserve in the long run? Inother words, what is the limit as t approaches infinity?c.) how many years will it take until there are 950 owls in the wildlife preserve? PLEASE HELP 3x^2 - 3x - 18 What is a typical development cycle using a Version Control System (VCS)?(1 point)Start a new branch, merge every functional version of the new feature as it is developed, send a pull request to the team and commit new features into the project.Build a repository, upload files, publicize the project, evaluate proposed changes and merge approved changes into the project.Send a pull request to the team, pull a new branch, commit every functional version of the new feature as it is developed and merge new features into the project.Start a new branch, commit every functional version of the new feature as it is developed, send a pull request to the team and merge new features into the project. Each gallon of gas cost $2.50. Nathan spent $30 on gas. Which value of x represents the number of gallons of gas Nathan purchased? If the period of a certain wave (wavelength = 4.5 m) is 2 seconds, what is the speed of the wave?1) 0.44 m/s2) 1.1 m/s3) 9.0 m/s4) 2.3 m/s